Evaluate the expression (-243)1/5

Answers

Answer 1

[tex]~\hspace{7em}\textit{rational exponents} \\\\ a^{\frac{ n}{ m}} \implies \sqrt[ m]{a^ n} ~\hspace{10em} a^{-\frac{ n}{ m}} \implies \cfrac{1}{a^{\frac{ n}{ m}}} \implies \cfrac{1}{\sqrt[ m]{a^ n}} \\\\[-0.35em] ~\dotfill\\\\ (-243)^{\frac{1}{5}}\implies \sqrt[5]{(-243)^1}\implies \sqrt[5]{(-3)(-3)(-3)(-3)(-3)} \\\\\\ \sqrt[5]{(-3)^5}\implies -3[/tex]

Answer 2
easy calculation : the expression evaluates to -243/5, or -48 3/5, and -48.6.

Related Questions

Find the value of x in the figure

Answers

Answer:

x=40º

Step-by-step explanation:

A straight line is 180º

180-40=140º

180-140=40º

x=40º

Hope this helps :)

Questions with Bounds:

1) Andy Mann is building a garden fence out of planks. The planks are 10cm wide to the nearest cm. He needs to make 75 m of fencing altogether, correct to the nearest meter. How many planks does he need to order to make sure he has enough to complete the fence?

Answers

A total of 785 planks Andy Mann needs to order to make sure he has enough to complete the fence.

As the planks are 10cm wide to the nearest cm.

The total length of the fencing is = 75 m

From above, we can conclude that,

Range of the planks' lengths: 9.5 cm <= Plank's length < 10.5 cm

Range of fencing needed: 74.5 m <= Fence length < 75.5 m

We should consider the worst-case scenario where all the planks are 9.5 cm.

To complete the job, he only needs 74.5 m in total.

So, the total number of planks needed for completing the Job will be

= (74.5 * 100) / 9.5 = (7.84210*100) = 784.210

For more questions on Range of length

https://brainly.com/question/24223676

#SPJ4

The scattered plot below shows the relationship between the total calories a meal has and the number of calories from fat.

If a meal has 300 calories from fat, how many total calories would you expect it to have?

Answers

Total 400 calories is expected to have a meal has 300 calories from fat  the relationship between the total calories a meal has and the number of calories from fat .

what is scatterplot ?

A scatter plot is a collection of dots that have been plotted along a horizontal and vertical axis. Statistics uses scatter plots because they can display the degree of connection, if any, between values of observable quantities or events (called variables). There are points on a scatter (XY) plot that depict the connection between two sets of data. The sample illustrates the weight and height of one person for each dot. Data points are plotted on a horizontal and vertical axis using scatter plots in an effort to demonstrate the degree to which one variable is influenced by another. The values of the columns selected on the X and Y axes determine where each row in the data table is represented by a marker, whose position.

given

300 calories from fat

total = 100 + 100 + 200

= 400 calories

Total 400 calories is expected to have a meal has 300 calories from fat  the relationship between the total calories a meal has and the number of calories from fat .

To know more about scatterplot visit :-

https://brainly.com/question/30017616

#SPJ1

City Fitness, a popular gym, currently has 840 members. Based on its membership statistics since opening a few years ago, City Fitness expects to grow its membership by a factor of
1
4
each year.
Write an exponential equation in the form y=a(b)x that can model the number of members, y, in x years.

Answers

The growth in x years for the gym will be represented as [tex]y = 840(1.25)^x[/tex].

What is an expression?

The mathematical expression combines numerical variables and operations denoted by addition, subtraction, multiplication, and division signs.

Mathematical symbols can be used to represent numbers (constants), variables, operations, functions, brackets, punctuation, and grouping. They can also denote the logical syntax's operation order and other properties.

City Fitness, a popular gym currently has 840 members. Based on its membership statistics since opening a few years ago, City Fitness expects to grow its membership by a factor of 1/4.

The expression can be written as,

[tex]y = 840(1.25)^x[/tex].

To know more about an expression follow

https://brainly.com/question/11737323

#SPJ1

Select the correct answer from the drop-down menu. You are conducting a survey on the best beaches in the world. The group that would probably give you the most reliable data is .
A) tourists who wish to visit the beaches
B) officials who manage the beaches
C) tourists who have visited the beaches
D) friends and family

Answers

The group that would probably give you the most reliable data is tourists who have visited the beaches. Option C is the correct answer.

In order to gather information about a service, product, or process, a survey is described as an act of looking at a process or interviewing a predetermined sample of people. Surveying beachgoers who have previously been there will yield more accurate results regarding the greatest beaches because they can share their personal experiences.

Surveying those in charge of the beaches is not a smart idea since they would be prejudiced in saying that their beaches are the finest. As a result, the third choice is the best one.

To learn more about the survey, visit the link below:

brainly.com/question/29205040

#SPJ4

The Burj Khalifa building in Dubai, United Arab Emirates, is one of the tallest buildings in the world at 2,722 feet tall. An American football field is 360 feet in length. Approximately how many football fields tall is the Buri Khalifa building? Round to the nearest tenth.​

Answers

Answer:

7.5 which rounds to 8

Step-by-step explanation:

2,722 divided by 360 equals 7.56

check if needed <3

(NEED HELP ASAP!!)

What are the coordinates of the focus of the parabola?


A: (8, 6)

B: (-8, 6)

C: (-8, 2)

D: (8, 2)

Answers

Answer:

C:  (-8, 2)

Step-by-step explanation:

Given parabola:

[tex]y=-\dfrac{1}{8}x^2-2x-4[/tex]

For the quadratic equation in the form y = ax² + bx + c, the x-value of the vertex is -b/2a.  

Therefore, the x-value of the vertex of the given parabola is:

[tex]\implies x=-\dfrac{b}{2a}=-\dfrac{-2}{2\left(-\frac{1}{8}\right)}=-8[/tex]

To find the y-value of the vertex, input x = -8 into the given equation:

[tex]\implies y=-\dfrac{1}{8}(-8)^2-2(-8)-4=4[/tex]

Therefore, the vertex (h, k) of the parabola is (-8, 4).

The focus of the parabola is (h, k+p) where:

(h, k) is the vertex[tex]p=\dfrac{1}{4a}[/tex]

Therefore:

[tex]\implies \textsf{Focus}=\left(-8,4+\dfrac{1}{4\left(-\frac{1}{8}\right)}\right)=\left(-8,2)[/tex]

Which transformation produces another triangle that has the same area as the one shown

Answers

A reflection in the line y = -2 and then translation of 2 units right will create the same area so option (C) will be correct.

How to plot a graph?

f we shift the triangle anywhere then there will be no change in the area but if we apply the scale factor then since the dimension changes so its area will also change.

In all options, there is a scale factor so it will change area but in option C

A reflection in the line y = -2 reflection gives a mirror image about y = -2 but no changes in area then translation of 2 units right will shift triangle 2 units right but no effect in the area.

Hence "A reflection in the line y = -2 and then translation of 2 units right will create the same area".

To know more about graph of function, check out:

https://brainly.com/question/24335034

#SPJ1

The total number of books sold from 2010-2015 from
Amazon can be modeled by the function
3
2
f(x) = 40x + 8x + 14824x + 33000and the number of kinds
of books at Amazon from that same time period can be modeled
as g(x): =
10x + 22, where x is the number of years since
2010. What is the average number of each kind of book
Amazon sold?

Answers

Answer:

If you want to find out how many print copies the book is selling, just click the Paperback or Hardcover edition of the book on Amazon, scroll down to Product Details on that page, and find the Amazon Best Sellers Rank for the print edition.

Step-by-step explanation:

What is another word for undecidable?

Answers

Therefore , the solution of the given problem of algorithm comes out to be incomputable. is another word for undecidable.

Clarify algorithm.

A calculation or problem-solving procedure is known as an computation or algorithm. Algorithms serve as a thorough set of instructions that direct hardware- or software-based routines through a sequence of planned actions step by step. In all areas of IT, algorithms are commonly employed.

Here,

If it is impossible to develop an algorithm that reliably returns a true or false result for each input value, the problem is said to be undecidable.

The category of inherently unknowable questions, a subtype of insoluble problems, only includes questions that ought to receive a yes-or-no response .

Thus ,incomputable. is another word for undecidable.

Therefore , the solution of the given problem of algorithm comes out to be incomputable. is another word for undecidable.

To know more about algorithm , visit:

https://brainly.com/question/29289479

#SPJ4

Mrs. Hanover borrows $1,413 at a
rate of 5% per year. How much
Simple Interest will she owe if it
takes her 9 months to repay the
loan? Round your answer to the
nearest cent

Answers

well, let's recall that a year has 12 months, so 9 months is really 9/12 of a year

[tex]~~~~~~ \textit{Simple Interest Earned} \\\\ I = Prt\qquad \begin{cases} I=\textit{interest earned}\\ P=\textit{original amount deposited}\dotfill & \$1413\\ r=rate\to 5\%\to \frac{5}{100}\dotfill &0.05\\ t=years\to \frac{9}{12}\dotfill &\frac{3}{4} \end{cases} \\\\\\ I = (1413)(0.05)(\frac{3}{4}) \implies I = 1413(\frac{3}{80})\implies I\approx 52.99[/tex]

The picture shows a shed. Find the length, AB, of the roof.​

Answers

The length AB of the roof is 3.20 m.

What is the Pythagorean theorem.

The Pythagorean theorem is one that can be used to determine the value of the unknown side of a right triangle when given two of its sides.

The theorem states that;

/Hyp/^2 = /Adj/^2 + /Opp/^2

Considering the given picture of the shed, draw a perpendicular to B from a point C on parallel line A. So that the length of AB can be referred to as the hypotenuse of triangle ABC.

Now applying the Pythagorean theorem, we have;

/Hyp/^2 = /Adj/^2 + /Opp/^2

/AB/^2 = /2/^2 + /2.5/^2

            = 4 + 6.25

/AB/^2 = 10.25

AB = (10.25)^ 1/2

AB = 3.20

Thus, the length of AB of the roof is 3.20 m.

Learn more about the Pythagorean theorem at https://brainly.com/question/231802

#SPJ1

11c+ 8) (-9c - 1)
Help plssss

Answers

Answer:

Step-by-step explanation: (11c + 8) (-9c - 1)

Then expand

which would be 11c ( -9c - 1) + 8 ( -9c - 1)

-99c^2 -11c - 72c - 8

It becomes -99c^2 - 83c -8

I hope this was helpful

Simplify:-
[tex]x {}^{2} - 5x + 6 \div x {}^{2} +3x - 10 \\ [/tex]
Guys help me please. ​

Answers

Step-by-step explanation:

[tex] = {x}^{2} \div {x}^{2} + 3x - 5x + 6 - 10 \\ = 1 - 2x - 4 \\ = - 2x - 4 + 1 \\ = - 2x - 3 \\ = - 1(2x + 3)[/tex]

HI,HOPE THAT IS HELPFUL.

Determine if the triangles can be proved congruent by SSS, SAS, ASA, AAS, or HL.
SSS

Answers

The triangles can be proved congruent by SAS theorem.

Properties of congruent triangles.

Two or more triangles are said to be congruent if and only if their corresponding length of sides and measure of internal angles are equal. Therefore, congruency is a property of equality of the triangles.

To prove that two or more triangles are congruent, then their corresponding sides are compared if they are equal. Also the measure of their corresponding internal angles are considered if they are equal.

Therefore. in the given question, to determine if the triangles can be proved congruent by the appropriate theorem;

It can be seen that the two sides of the triangles are equal. Also the measure of their included angles are equal. Therefore, the triangles can be proved to be congruent by SAS.

Learn more about the properties of congruent triangles at https://brainly.com/question/3999145

#SPJ1

Find the lope of a line parallel to the line whoe equation i 6xy=-36xy=−3. Fully implify your anwer

Answers

The line parallel to 6y = -36x -3 will have the same slope which will be -6.

What is Slope of a line ?

A line's steepness and direction are measured by the line's slope. Without actually using a compass, determining the slope of lines in a coordinate plane can assist in forecasting whether the lines are parallel, perpendicular, or none at all.

The increase divided by the run, or the ratio of the rise to the run, is known as the line's slope. In the coordinate plane, it describes the slope of the line. Finding the slope between two separate locations and calculating the slope of a line are related tasks. In general, we require the values of any two separate coordinates of a line in order to determine its slope.

Any two different points on a line may be used to compute the slope of any line. The ratio of "vertical change" to "horizontal change" between two different locations on a line is calculated using the slope of a line formula.

The equation of the line is

6y = -36x -3

⇒y = -6x - 1/2

From slope intercept form of the equation we know that

equation of the line is y = mx + c

where m is the slope.

Therefore,

slope the line is -6

The line parallel to 6y = -36x -3 will have the same slope which will be -6.

To learn more about Slope refer to :

https://brainly.com/question/29735037

#SPJ4

a) What is the size of angle ABC? b) Use your answer to part a) to work out the size of reflex angle CBD. Give your answers to the nearest degree. ​

Answers

Answer:

a) 110

b) 250

Step-by-step explanation:

Read the top row of numbers for a.  We are starting at zero and going to 110

Reflex is when two angles add up to 360

360 - 110 = 250

What are the 3 steps to solving a two step equation?

Answers

Answer:

Use the following denomination (Ghana cedis 200,100,50 and 20)to find the quantities of notes

1.3584655000

2.85760000

3.146737000

4.5555627000

5.7777000

please help

Step-by-step explanation:

1. Eliminate any constant that is on the same side as the variable.

2. Use inverse operations to isolate the variable by itself.

3. Whatever you do to one side you do to the other.

hat is the product of the least common multiple and the greatest common factor of $22$ and $48$?

Answers

Answer:

  1056

Step-by-step explanation:

You want the product of the LCM and the GCF of 22 and 48.

Product

One way to find the LCM of two numbers is to start with their product, then divide out the extra factor:

  LCM(A, B) = (A×B)/GCF(A, B)

Since we want the product of the LCM and the GCF, we can start with this and multiply by the GCF:

  LCM(A, B) · GCF(A, B) = ((A×B)/GCF(A, B)) · GCF(A, B)

  LCM(A, B) · GCF(A, B) = A×B

Using this for the given numbers, we get ...

  LCM(22, 48)·GCF(22, 48) = 22·48 = 1056

__

Additional comment

If you want to go to the trouble of finding the LCM and GCF, you get ...

  22 = 2·11

  48 = 2·24

  LCM(22, 48) = 2·11·24 = 528

  GCF(22, 48) = 2

  LCM·GCF = 2·528 = 1056

Write the equation of the line that passes through (7, 6) and (-- 1, 2) in slope-intercept form.
○ y = − 1/x + 2/1/2
O y = 2x + 4
○ y = 1/x + 1/2/2
○ y = 1/x + 1/2

Answers

The equation of the line that passes through (7, 6) and (-1, 2) in slope-intercept form is, [tex]y=\frac{1}{2}x + \frac{5}{2}[/tex].

What is slope intercept form of the line?

Using a straight line's slope and the location of its y-axis intersection, the slope intercept equation can be used to determine the general equation of the line. y = mx + b is the equation in slope intercept form.

Given:

The line passes through the points (7, 6) and (- 1, 2).

We have to find the equation of the line that passes through (7, 6) and (-- 1, 2) in slope-intercept form.

First to find the slope of the line using the formula,

Slope = m = [tex]\frac{y_2-y_1}{x_2-x_1}[/tex]

Here, [tex](x_1, y_1)= (7, 6), (x_2,y_2)=(-1, 2)[/tex]

Plug the values in the above formula.

[tex]m = \frac{2-6}{-1-7} = \frac{-4}{-8}=\frac{1}{2}[/tex]

Now consider the point slope form of the line,

[tex]y-y_1=m(x-x_1)\\y-6=\frac{1}{2}(x-7) \\y-6=\frac{1}{2}x-\frac{7}{2}\\ y=\frac{1}{2}x -\frac{7}{2}+6 \\y=\frac{1}{2}x+\frac{5}{2}[/tex]

Hence, the equation of the line that passes through (7, 6) and (-1, 2) in slope-intercept form is, [tex]y=\frac{1}{2}x + \frac{5}{2}[/tex].

To learn more about slope-intercept form, of the line click on the link

https://brainly.com/question/30094629

#SPJ1

Lloyd's Llamas has $3,241.77 in liabilities and $12,457.12 in owner's equity. If the accounting equation is in balance, Lloyd's has $_______ in assets.

Answers

The number of asset is 15.698.89

How to calculate the number of asset ?

Assets = liabilities + equity

liabilities= 3,241.77

equity= 12,457.12

Assets= 3,241.77 + 12,457.12

=  15.698.89

Hence Lloyd has 15.698.89 number of assets

Read more on assets here

https://brainly.com/question/25646730

#SPJ1

A space ship travels at 57 miles per hour and traveled for 521 hours.
Select the answer that represents how many miles the space ship traveled.

Answers

Answer:29,697

Step-by-step explanation: multiply 57x521 and you get 29,697

The spaceship traveled 29,697 miles.

Please hurry its timed and i really need help!

Answers

Hello!:

Let's take this problem in steps:

   [tex]sinx-\sqrt{3-3sin^2(x)}=0\\\\\text{We know that } sin^2(x) + cos^2(x)=1 \text{ and that } cos^2x=1-sin^2(x)\\\\sin(x)-\sqrt{3(1-sin^2(x))}=0\\\\sin(x)-\sqrt{3*cos^2(x)}=0\\\\sin(x)-\sqrt{3}*cos(x)=0[/tex]

Now we need to find the value of sin(x) and cos(x) where the value becomes zero:

  --> and it so happens to be:

        [tex]sin(30)=\dfrac{1}{2} \\\\cos(120)=\dfrac{\sqrt{3}}{2}[/tex]

Thus:

 [tex]sin(30)-\sqrt{3}*cos(120)=0\\\\\dfrac{1}{2} -\sqrt{3}*\dfrac{\sqrt{3} }{2} =0\\\\\\\dfrac{1}{2} -\dfrac{1}{2} =0\\\\0=0[/tex]

Answer: (B)

Tank 1: 36 inches long, 18 inches wide, and 12 inches tall. How many gallons of water is in the tank?


I need a step-by-step explanation please

Answers

In a tank that is 36 inches long, 18 inches wide, and 12 inches tall have approximately 33 gallons of water.

How to calculate volume of a tank?

The rectangular tank's volume may be calculated using the formula V = l*b* h, where "l" stands for the tank's length, "b" for its width, "h" for its height, and "V" for its volume.

Define volume.

The space occupied within an object's borders in three dimensions is referred to as its volume. It is sometimes referred to as the object's capacity.

Volume = length*width*height

V=36*18*12

V=7776 inches^3

which is approximately 33 gallons.

To learn more about the volume visit the link:

https://brainly.com/question/463363

#SPJ4

Out of 30 students surveyed, 17 have a dog. based on these results, predict how many of the 300 students in the school have a dog?

Answers

Answer: 170 students

1. Use the proportion 17/30 = x/300 to find the answer

2. Cross multiply
30x = 5100, which can also be written as 3x = 510 because they both have at least 1 zero and 0/0 cancels out.

3. Divide both sides by 3 to get the variable alone
X = 3x/3 = 510/3

4. Simplify
X = 170

A container has cups of milk in it. The container is 1/3 full. How many cups does the container hold? Explain or show your reasoning.

Answers

Answer:

Step-by-step explanation:

2 or 6 i think im probaly wrong

Answer:

The container can hold 6 cups.

Step-by-step explanation:

Since the container is 1/3 full, and it currently has 2 cups, we know that it can only hold 6 cups as 3 × 2 is equal to 6.

help. pls.
-photo attached !

Find the measures of the numbered angles.


(the angles i need to find)

<1 _?
<2 _?
<3 _?
<4 _?
<5 _?
<6 _?
<7 _?
<8 _?

Answers

Answer:

<1=75°[corresponding angles are equal]

<2=180-<1=(180-75)°=105°[sum of st.angles is supplementary]

<3=<2=105°[Vertically opposite angles are equal]

<4=75°[Alternate angles are equal]

<6=<2=105°[Corresponding angles are equal]

<7=<6=105°[Vertically opposite angles are equal]

<8=75°[Vertically opposite angles are equal]

*mark me brainliest

somebody help please

Answers

Answer:

Step-by-step explanation:

because HA⊥HR, HR⊥DR, so HA ║ DR, then we know ∠DHR=∠ARH,

we konw, AR=HD, They have a common segment HR, as well as the same angle,so HRA≅RHD

How can you tell if you have a perfect square trinomial or the DIFFERENCE OF SQUARES?

Answers

The value of a perfect square is either a positive number or zero. They cannot be neutral.

what is perfect square ?

Perfect squares are generated when you square an integer. For instance, the number 81 is a perfect square since it can be derived by squaring 9: 99=81. 144 is a perfect square since 12 is squared to produce it. 169 is a perfect square since 13 may be squared to obtain it. Mathematical language Informally: When an integer (a "whole") number, whether positive, negative, or zero, is multiplied by itself, the outcome is referred to as a square number, a perfect square, or simply "a square." Therefore, all square numbers are 0, 1, 4, 9, 16, 25, 36, 49, 64, 81, 100, 121, 144, and so on.

given

The value of a perfect square is either a positive number or zero. They cannot be neutral.

Square differences can be positive, negative, or zero depending on the order in which the numbers are obtained.

One squared term will be added to the difference of squares, and another squared term will be subtracted.

To know more about perfect square visit :-

brainly.com/question/385286

#SPJ4

find the volume of the given solid. bounded by the planes z = x, y = x, x + y = 5 and z = 0

Answers

The volume of the solid given is 125/24 and is constrained by the planes z = x, y = x, x + y = 5, and z = 0.

what is volume ?

The capacity of an object is expressed in terms of volume. The quantity of space a three-dimensional item takes up can also be referred to as volume.  It is sometimes referred to as the vessel's "capacity" or "volume." infant milk bottle with milliliter measuring indications and juice bottle with a 1 liter capacity.

Given

The bounds for z are 0 ≤ z ≤ x.

The bounds in the xy-plane are x ≤ y ≤ 5 - x and x in [0, 5/2]

So, V = ∫∫ (x - 0) dA

= ∫(x = 0 to 5/2) ∫(y = x to 5 - x) x dy dx

= ∫(x = 0 to 5/2) x (5 - 2x) dx

= ∫(x = 0 to 5/2) (5x - 2x^2) dx

= (5x^2/2 - 2x^3/3) {for x = 0 to 5/2}

=  125/24.

The volume of the solid given is 125/24 and is constrained by the planes z = x, y = x, x + y = 5, and z = 0.

To know more about volume visit :-

https://brainly.com/question/13338592

#SPJ1

Other Questions
Please help I have a learning disability and this is due todayDetermine whether the two expressions are equivalent. If so, tell what property is applied. If not, explain why. 0 + 32 and 0yes, it uses the Commutative PropertyNo they are not equivalent. The first expression is equal to 32, not 0.yes, it uses the Associative Propertyyes, it uses the Identity Property Is 0 the middle of a number line? Who held the power of government under the Articles of Confederation? PLS HELP ASAP 50 POINTS!!!Identify or mark the missing side or angle that would make triangle ABC congruent to triangle PDF by AAS Which of the following statements regarding the disaster loss treatment of retirement distributions is FALSE?Form 8915-E is used to report the distribution, any repayments, and figure the taxable amount, if any, of qualified disaster distributions.An individual doesn't have to live in the qualified disaster area during the disaster or have sustained economic loss to take a qualified disaster distribution from a retirement plan.Qualified disaster distributions have a limit of $100,000 in the aggregate per qualified disaster.The 10% tax (penalty) on early distributions of retirement funds does not apply to any qualified disaster distribution. Given U(-6, 7), V (1, -7), W (6, 4), and X (10, y). Find y such thatUV LWX. Determine whether the geometric series is convergent or divergent. 10 8 + 6.4 5.12 +... A. convergent B. divergent If it is convergent, find its sum. (If the quantity diverges, leave this blank.) _____ What is the measure of m?nm287m = [? ] VGive your answer in simplest form. Suppose you were going to create a map of the sea floor where the Christmas earthquake of 2004 occured for the US Government. How would you proceed? What methods would you use and why? When drafting your answer, consider the depth and contours of the sea-floor. One of the roots of the equation x^2-5x+q=0 is 2. Find the other root and the value of the coefficient q. To be eligible for the swim team, the mean pace a student needs to achieve on 5 laps of the pool is 60 seconds (s). The student swims a lap in 63 s, 62 s, 65 s, and 58 s on her first four laps.What is the maximum time (in seconds) that she can take on her last lap and still make the team? What are the factors needed to consider in buying and selling products and services? F(x) = 3x -4 find each value of F(0) Simplify the following polynomial, then evaluate for x = -22x2 - 4x + 3x2 + x - 7 What is the area of the circle if BC = 6cm? how would you characterize the difference between the way Othello and Iago talk, both in their subject matter and their style What is a fixed and variable expense and provide an example of each? In testing inventory at an audit client in the retail industry, you note that some of the inventory is contracted to be held on consignment. As a result, which financial statement assertion is now relevant ?a. Rights and obligations.b. Completeness.c. Existence or occurrence.d. Valuation or allocation. What do snails and clams have in common? Part I-Selected Response1. CJ wrote several pairs of values that hethought were equivalent. Which of hisequations are correct? Circle all that apply.A. 1.25-125%. B. 0.03-30%C. 0.001 - 1% D. E. -50% 2. What is 30% of 60?A. 18B. 20C. 24D. 30B. 0.03-30%D. -60%F.D. 200 3. Six is 3% of what number?A. 0.18B. 2C. 18 4. What percent of 40 is 60?A. 20%B. 50%C. 67%D. 150%5. What is the percent decrease from 20 to16?A. 4%B. 20%C. 25%D. 125% 6. Last year, 15 dogs entered the local dogshow. This year, 20 entered. What is thepercent increase?A. 20%B. 25%C. 33%D. 75% 7. Roe bought a video game in Miami. Thegame was $30. Miami has a 7% sales tax.How much money did Roe pay for the videogame including sales tax?A. $32.10B. $33.50C. $37.00D. $51.00 8.Lia bought a new jacket. The original priceof the jacket was $48 but it was on sale for15% off. For numbers 8a-8d, determinewhether each statement about Lia'spurchase is true or false. 8a. Lia paid more than $48for the jacket. 8b. The discount was $7.20. 8c. Lia paid $40.80 for thejacket. 8d. Lia paid $55.20 for thejacket.8a. TRUE FALSE8b. TRUE FALSE8c. TRUE FALSE8d. TRUE FALSE